Discussion Forum

Need help with an AMC8 mock question

 
 
LiuAnna的头像
Re: Need help with an AMC8 mock question
LiuAnna - 2017年10月27日 Friday 11:54
 

2, 3, 5, and 7 are prime numbers. The prime factorization of 588 is 2^2 x 3 x 7^2. 3= 3^1, and since there is no five, you write it as 5^0. 2(2) + 3(1) + 5(0) + 7(2) = 4 + 3 + 0 + 14 = 21, so the answer would be (a) 21.